Both conservative and solenoidal vector field

Click For Summary
SUMMARY

A vector field A that is both solenoidal and conservative can be expressed as A = -∇Φ, where Φ is a scalar function. The discussion clarifies that if A is solely a conservative vector field, it is represented as A = ∇φ, leading to confusion regarding the negative sign. The requirement for A to be solenoidal does not alter the fundamental representation of A as a gradient of a scalar function, but it emphasizes the divergence-free condition of the vector field.

PREREQUISITES
  • Understanding of vector calculus, specifically gradient and divergence operations.
  • Familiarity with the concepts of conservative and solenoidal vector fields.
  • Knowledge of scalar functions and their gradients.
  • Basic principles of mathematical physics related to vector fields.
NEXT STEPS
  • Study the properties of solenoidal vector fields in detail.
  • Learn about the implications of vector fields being conservative in physics.
  • Explore the mathematical derivation of the relationship between scalar functions and vector fields.
  • Investigate examples of solenoidal and conservative vector fields in applied contexts.
USEFUL FOR

Students of physics and mathematics, particularly those studying vector calculus and field theory, as well as educators seeking to clarify concepts related to vector fields.

aruna1
Messages
110
Reaction score
0

Homework Statement



if a vecor A is both solenoidal and conservative; is it correct that

A=-▼Φ

that is

A=- gradΦ

Φ is a scalar function

thanks
 
Physics news on Phys.org
If A is just a conservative vector field, then A= -\nabla \phi for some scalar function \phi. I'm not sure what requiring that it also be solenoidal adds.
 
HallsofIvy said:
If A is just a conservative vector field, then A= -\nabla \phi for some scalar function \phi. I'm not sure what requiring that it also be solenoidal adds.

well i thouhgt if A is only conservative
then

A= \nabla \phi (according to my textbook)

not

A= -\nabla \phi

so what is right?:confused:
 
Question: A clock's minute hand has length 4 and its hour hand has length 3. What is the distance between the tips at the moment when it is increasing most rapidly?(Putnam Exam Question) Answer: Making assumption that both the hands moves at constant angular velocities, the answer is ## \sqrt{7} .## But don't you think this assumption is somewhat doubtful and wrong?

Similar threads

  • · Replies 5 ·
Replies
5
Views
2K
  • · Replies 1 ·
Replies
1
Views
3K
  • · Replies 2 ·
Replies
2
Views
2K
  • · Replies 4 ·
Replies
4
Views
2K
  • · Replies 3 ·
Replies
3
Views
3K
Replies
12
Views
2K
Replies
5
Views
2K
  • · Replies 5 ·
Replies
5
Views
2K
Replies
4
Views
1K
  • · Replies 2 ·
Replies
2
Views
1K